Download as pdf or txt
Download as pdf or txt
You are on page 1of 10

Suggested Solution for Applied Mathematics 2006 (I)

Paper I 4a + 3b = 12

r r r r r r 12 + b = 12 , gives a = 3 , b = 0
1. (a) AB = 8i , BD = −8i + 6 j , VC = OC − OV = 4i + 3 j − 12k 9 − a = 6

r r r r r
Let the forces act along AB , BD and OV be F1 , F2 and F3 . i.e. OP = 4i + 3 j

r AB r r BD r r
F1 = 8 = 8i , F2 = 10 = −8i + 6 j
| AB | | BD | 2. (a) For interception occurs, the path of relative velocity should lie along P0 Q0
r BD r r r By cosine law,
F3 = 13 = 4i + 3 j − 12k
| BD | (ku ) 2 = u 2 + v 2 − 2uv cos 60 o P0
r
3r r r r i.e. v 2 − uv + (1 − k 2 )u 2 = 0 −v P (u)
Resultant force = ∑ Fi = 4i + 9 j − 12k 30 o
i =1 (b) For interception occurs with 2 distinct
r r
(b) Resultant moment about O directions, we should have Q vP
vQ (ku)

r r r two positive and distinct values of v. (v)


= OA × F1 + OB × F2 + OV × F3 2 2 2 2
Q0
* 0 < ∆ = u − 4(1 − k )u = u (4k − 3)
r r r r r r
i j k i j k 3
r k> (since k > 0 )
= 4 −3 0 +0+ 0 0 12 2
8 0 0 4 3 − 12 * 0 < product of roots of v = (1 − k 2 )u
r r r
= − 36i + 48 j + 24k 0 < k <1
(c) Let the coordinates of P be (4, a, b). 3
i.e. < k <1
Comparing the rotating effect about O: 2
r r r
i j k
r r r r r r
− 36i + 48 j + 24k = OP × (4i + 9 j − 12k ) = 4 a b
4 9 − 12
r r r
= (−12a − 9b)i + (48 + 4b) j + (36 − 4a )k

Page 1
Suggested Solution for Applied Mathematics 2006 (I)
3. (4 + 3 )W (4 − 3 )W
B R (b) When θ = 30 o , N 1 = , N2 =
C 2 4
G N2
N1 R W
F1 = F2 =
θ 2
O A
F1 F2 F1 1 1
2W W Consider = < =µ
N1 4 + 3 2

OC r OC r F2 2 2 1
Note: AG = 2r , AC = = , OA = = = > = =µ
tan θ tan θ sin θ sin θ N2 4 − 3 4 2

(a) The definitions of forces are skipped here. i.e. Equilibrium cannot be occurred.
For the whole system
u
Resolving forces along horizontal and vertical directions: 4. (a) Let u and v be the speed just before and
 F1 = F2 after impact with the inclined plane.
45 o

 N 1 + N 2 = 3W * By conservation of linear momentum
For the rod along plane: v α
Take moment about A: v cos α = u cos 45 o 45 o

r * By Newton’s law of restitution:


(2r )W cos θ = R, ∴ R = 2W sin θ
tan θ v sin α = e(u sin 45 o )
Resolving forces along horizontal direction: Divides two equations, we have e = tan α
F2 = R sin θ = 2W sin 2 θ 1+ e2 2 1+ e2
(b) v 2 = (u cos 45 o ) 2 + (eu sin 45 o ) 2 = u = (2 gh) = gh(1 + e 2 )
Resolving forces along vertical direction: 2 2
N 2 + R cos θ = W , N 2 = W − 2W sin 2 θ 1 1 mgh
Energy loss = mgh − mv 2 = mgh − mgh(1 + e 2 ) = (1 − e 2 )
2 2 2
Upon solving, we have N 1 = 2W (1 + sin θ cos θ) (c) change in momentum of P along normal
2
N 2 = W (1 − 2 sin θ) = m(v sin α) − m(−u sin 45 o ) = m(eu sin 45 o ) + mu sin 45 o
F1 = F2 = 2W sin 2 θ
= m gh (1 + e)
R = 2W sin θ

Page 2
Suggested Solution for Applied Mathematics 2006 (I)
5. (a) Let a be the acceleration of the particle moving up the plane PC. u 2 cos λ
t2 =
Under motion, F = (tan λ) R . g sin α sin(α + λ)
Resolving forces perpendicular and along plane: a t1 sin α cos λ
R Upon division, we have =
 R = mg cos α t2 sin( α + λ)

− F − mg sin α = ma F 6. (a) Equate vertical forces:
R T
(tan λ) R α mg T sin 30 o + R sin 60 o = mg
a=− − g sin α
m T + 3R = 2mg --- <1> l sin 60 o
= − tan λ ( g cos α + g sin α) Equate horizontal forces:
mg
g g sin(α + λ) u2
=− (sin λ cos α + cos λ sin α) = − T cos 30 o − R cos 60 o = m
cos λ cos λ l sin 60 o
By “ v 2 − u 2 = 2as ”, 4mu 2
3T − R = --- <2>
2 2 g sin(α + λ) l 3
0 − u = 2( − ) AC
cos λ 4 3mu 2
< 1 > × 3− < 2 > : 4 R = 2 3mg −
u 2 cos λ 3l
i.e. AC =
2 g sin(α + λ) g u2
i.e. R = 3m ( − )
(b) For the particle moves up the plane: 2 3l

g sin( α + λ ) (b) Let u ' be the horizontal speed of the particle at time t.
By “ v = u + at ”, 0 = u − t1 ,
cos λ By conservation of angular momentum: (conserves, as no external torque
u cos λ along the tangent of the circle.)
t1 =
g sin(α + λ) l OP lu
m( o
)u = m( o
)u ' , u' =
As BC is smooth, the downward acceleration is g sin α . sin 60 sin 60 OP

1 2 1 Let R' be the normal reaction of the cone at time t.


By “ s = ut + at ”, BC = ( g sin α)t 2 2 ,
2 2 g u' 2 g l 2u 2 3mg l
R ' = 3 m( − ) = 3m ( − 3
)= (1 − )
u 2 cos λ g sin α 2 2 3OP 2 3(OP) 2 3(OP) 3
= t2
2 g sin( α + λ) 2 l
So the particle leaves the surface when R' = 0 , i.e. OP = .
3
3

Page 3
Suggested Solution for Applied Mathematics 2006 (I)

 x = ut
 y
7. (a) (i)  1 2 As the ball is perfectly elastic, the speeds along and perpendicular to the
 y = vt − 2 gt
 ground just after impact are u and v respectively.
v
Eliminate t, we have Set the x-y-plane at O' as shown in
x
v gx 2 u the figure. y
y= x− 2 h
u 2u In order to fail to clear the second wall,
O'
x
2uv h> y 3d
(ii) Put y = 0, x = R ,we have: Range = R = x=
g 2
3d
2
vs gs 2 u gs 2 v 3d g 3d
(b) d≤y = − , i.e. v≥ (d + 2 ) --- <*> = ( )− 2 ( )
x=s u 2u 2 s u 2 2
2u 2u

5 gd gd = 3d
(c) * Put v = , u , s = 2d into <*>
2 2
5 gd u gs 2 9 gd
LS = , RS= (d + 2 ) =
2 s 2u 4
Then LS > RS and hence the ball will clear the first wall.
* Let the ball first hit the ground at the point O' .
2uv 5d
OO' = =
g 2

* The distance between O' and the 2nd wall

3d
= 4d − OO' =
2

Page 4
Suggested Solution for Applied Mathematics 2006 (I)
8. (a) Equation of motion of P along radial direction: g g
x = A sin t + B cos t + 2l
r l l
mg eθ
m(&r& − rω 2 ) = T = e r
l er g g g
r x& = (− B sin t + A cos t)
g θ l l l
i.e. r&& − rω2 = e --- <1> O
l  x(0) = s(0) − r (0) = l − 0 = l
As 
Equation of motion of Q along radial direction:  x& (0) = s&(0) − r&(0) = 0 − 0 = 0
mg l = B + 2l
m(&s& − sω 2 ) = −T = − e 
l ∴  g , A = 0 , B = −l
0 = A
g  l
i.e. &s& − sω 2 = − e --- <2>
l g
x = −l cos t + 2l
(b) (i) Note that x = s − r and e = x − l l

d2 d 2g g
<2> – <1>: 2
(s − r ) − ω 2 (s − r ) = − e As − 1 ≤ cos t ≤ 1 , we have l ≤ x ≤ 3l .
dt dt l l

2g 2g
&x& − ω 2 x = − ( x − l) (iii) Put ω 2 = ,
l l

2g <*> becomes: &x& = 2 g ,


i.e. &x& + ( − ω2 )x − 2g = 0 --- <*>
l x& = 2 gt + C

g g As x& (0) = 0 , C=0


(ii) Put ω 2 = , <*> becomes: &x& + x − 2g = 0
l l ∴ x& = 2 gt

g x = gt 2 + D
∴ &x& = − ( x − 2l)
l As x(0) = l , D=l
2
which is a SHM i.e. x = gt + l
(with centre of oscillation at x = 2l )

Page 5
Suggested Solution for Applied Mathematics 2006 (I)
y
9. (a) By symmetry, y=0 (b) (i) Set the x-y-plane as shown in the figure. B
Coordinates of centre of mass of S
∫ xdm
x= a
∫ dm
y
= (0,
3
)

2a Coordinates of centre of mass of hemisphere x



= 0
x(ρπy 2 dx)
O
2a −3a A
∫0 (ρπy 2 dx) dx = (0,
8
)

12a Let coordinates of centre of mass of the toy be G( x, y ).


∫0
x[ ( x − 2a) 2 ] 2 dx
4
= x = 0 is obvious.
2a 1
∫0 [ 4 ( x − 2a) ] dx
2 2

a −3a
12W ( ) + 5W ( )
2a 3 8 =a
=
∫0 x( x − 2a) 4 dx y=
12W + 5W 8
2a
∫0 ( x − 2a) 4 dx
the position of the centre of mass of the toy
0 a
∫ (u + 2a)u
4
du is at a distance above the centre of the plane face of the
= −2a 0
(put x − 2a = u ) 8
∫−2a u
4
du
hemisphere.
0
∫ (u
5
+ 2au 4 )du
= − 2a 0
∫−2a u
4
du

0
u 6 2a 5 ( 2a ) 6
[ + u ]
6 5 a
= 0
− 2a
= 30 5 =
1 (2a ) 3
[ u5]
5 − 2a
5

Page 6
Suggested Solution for Applied Mathematics 2006 (I)
(ii) C θ 10. (a) Let the initial extension be e.
P
4mg l
e = mg , e=
R O l 4
G θ
l
(b) (i) At time t, the extension of the string is 2 x − y + .
4
A F B
So the tension (T) of the 3 portions of the extensible string:
17W
4mg l
T= (2 x − y + )
OA a 1 1 2 l 4
* tan θ = = = , ∴ sin θ = , cos θ =
OB 2a 2 5 5 Equation of motion of A: 3m&x& = 3mg − 2T --- <1>

a Equation of motion of B: m&y& = T − mg --- <2>


* OA = OC = a, OG =
8 4mg l
where T = (2 x − y + )
Resolving force horizontally and vertically: l 4
F = P and R = 17W (ii) (1) <1> + 2 × <2>: 3m&x& + 2m&y& = mg i.e. 3&x& + 2 &y& = g .
Take moment about O: 1
(2) From (b)(ii)(1), &x& = ( g − 2 &y&)
(OC ) P + (OA) F cos θ = (OA) R sin θ + (OG )17W cos θ 3

2 1 a 2 As x& (0) = y& (0) = 0 and x(0) = y (0) = 0 ,


aP + aF ⋅ = aR ⋅ + ⋅17W ⋅
5 5 8 5 g 2 g 2
we have x& = t − y& and x = t 2 − y .
5 3 3 6 3
( 5 + 2) P = 17W ⋅
4 T 4g l
By <2>: &y& = −g = (2 x − y + ) − g
85W 85( 5 − 2)W m l 4
P= (or P = )
4( 5 + 2) 4 4g 4g g 2 4
= (2 x − y ) = ( t − y − y)
l l 3 3

4 g 2 2 28
= t − y
3l 3l

28 4g 2 2
i.e. &y& + y= t --- <*>
3l 3l

Page 7
Suggested Solution for Applied Mathematics 2006 (I)

28 28 g 11. (a)
(iii) Consider the auxiliary equation: λ2 + = 0, λ = 0± i y b
3l 3l y= a2 − x2
a
y Ix
28 g 28 g
y h = A sin t + B cos t x
3l 3l

g 2 3l
yp = t −
7 98 dx
28 g 28 g g 3l Let I x and I y be the moment of inertia of the lamina about the x-
∴ y = A sin t + B cos t + t2 −
3l 3l 7 98 and y- axis respectively.

28 g 28 g 28 g 2g In the following proof, we make use of the fact that


y& = (− B sin t + A cos t) + t
3l 3l 3l 7 Moment of inertia of an uniform rod about it centre of mass and

 3l 1
(mass)( half length )2.
 y ( 0) = 0 0 = B − 3l perpendicular rod is
As  ,  98 , i.e. A=0, B= 3
 y& (0) = 0 0 = A 98
 1 b
Ix = ∫ dI = ∫ ( y 2 dm) ,
3
(where y =
a
a2 − x2 )
3l 28 g g 3l
y= cos t + t2 −
98 3l 7 98 1 2ρ 4πb 3 a
3
=
3 ∫ y 2 (ρ ⋅ 2 y ⋅ dx) =
3 ∫ y 3 dx =
3a 3
∫0 (a 2 − x 2 ) 2 dx

π
4ρab 3 2
=
3 ∫
0
cos 4 θdθ (put x = a sin θ )

π
2
Now, define J n = ∫ cos n θdθ
0
π
π
2
2
J n = [sin θ cos n −1
θ] 0 + (n − 1) ∫ cos n − 2 θ sin 2 θdθ
0

= (n − 1) J n − (n − 1) J n − 2

n −1
i.e. Jn = Jn
n

Page 8
Suggested Solution for Applied Mathematics 2006 (I)
π
Alternative Method
3 1 3 1 3π
Therefore, J 4 = ⋅ J 0 = ⋅ ∫ 2 dθ =
4 2 4 2 0 16 x2 y2
I 0 = moment of inertia of the ellipse + = 1 about an axis
4ρab 3
π
16h 2 9h 2
2
∴ Ix =
3 ∫
0
cos 4 θdθ
through O perpendicular to its plane

πρab 3 = ∫ dI
=
4

πρab 3 1 y2
Similarly, Iy = = ∫ ( x 2 dm + (dm) y 2 ) = ∫ ( x 2 + )dm
4 3 3

x2 y2 y2 4h y3
I 0 = moment of inertia of the ellipse 2
+ 2
= 1 about an axis = ∫ (x 2 + )(ρ ⋅ 2 y ⋅ dx) = 2ρ ∫ ( x 2 y + )dx
a b 3 −4h 3
through O perpendicular to its plane π
= 2ρ ∫ (48h 3 cos 2 θ sin θ + 9h 3 sin 3 θ)(4h sin θdθ)
= Ix + Iy 0

πρab(a 2 + b 2 ) (put x = 4h cos θ , y = 3h sin θ )


=
4 π π
= 8h 4 ρ(48∫ sin 2 θdθ − 39∫ sin 4 θdθ) [Refer to <*>]
0 0
πρ(4h)(3h)(25h 2 )
Put a = 4h , b = 3h , I0 = = 75πρh 4
4 π 3π
= 8h 4 ρ(48 ⋅ − 39 ⋅ ) = 75h 4 ρ
m 25 2 8
= 75πh 4 ⋅ 2
= mh 2
12πh 4 m 25mh 2
= 75h 4 ⋅ =
i.e. Required moment of inertia 12πh 2 4

89 i.e. Required moment of inertia


= I 0 + m(16h 2 ) = mh 2
4 89
= I 0 + m(16h 2 ) = mh 2
4

π n −1
<*>: Define K n = ∫ sin n θdθ , using by parts, we have K n = Kn−2 .
0 n

Page 9
Suggested Solution for Applied Mathematics 2006 (I)
(b) Required moment of inertia

89 1
= (ρ ⋅12πh 2 )h 2 − [ (ρπ(2h) 2 )(2h) 2 + ρπ(2h) 2 (4h) 2 ]
4 2
M M
= 195πh 4 ρ , where ρ = =
12πh 2 − π(2h) 2 8πh 2

195Mh 2
=
8
(c) Let G be the centre of mass of the signboard.

1
Then PG = PQ = 4h
2 X

Let ω be the required angular speed.


Y G
By conservation of energy: P
1 195Mh 2 2 ω
( )ω = Mg (4h)
2 8
G
64 g
i.e. ω= Mg
195h
Let X and Y be the reaction at P along upward vertical
and horizontal directions.
Y = 0 is obvious.
X − Mg = M (4h)ω 2

64 g
X = Mg + M (4h)( )
195h

451Mg
=
195

451Mg
Reaction = X 2 + Y 2 =
195

Page 10

You might also like